Как прецессия влияет на угловой момент?

Я изучал задачник Иродова. Есть много хороших задач, которые бросают вызов концептуальному знанию, поэтому я должен попросить кого-нибудь дать качественный ответ. Лучше всего я могу описать свое замешательство на примере, поэтому давайте возьмем задачу 1.284:

У нас есть гироскоп, прецессирующий вокруг своей вертикальной оси (рис. 1.73). Ось гироскопа горизонтальна.

Гироскоп шарнирно закреплен в точке О.

Мне удалось найти онлайн-решение здесь . Итак, поскольку гироскоп вращается вокруг своей оси симметрии, я согласен, что он будет иметь угловой момент в горизонтальной плоскости (у него есть радиальная угловая скорость).

Однако кажется, что это решение не учитывает угловую скорость прецессии, поскольку она связана с угловым моментом гироскопа. Если гироскоп вращается в горизонтальной плоскости, то вектор угловой скорости прецессии должен указывать вертикально. Не должен ли тогда гироскоп иметь угловой момент в вертикальном направлении?

Может быть, это связано с системой отсчета (может быть, мы наблюдаем гироскоп из системы отсчета, вращающейся вертикально с угловой скоростью, равной угловой скорости прецессии)...? Если это так, то почему нет центробежной силы?

Что-то все равно не сходится.

Ответы (4)

Вводные книги всегда предполагают, что ю много больше угловой скорости прецессии. В этом случае задача упрощается тем, что мы можем пренебречь угловым моментом, обусловленным прецессией. Если вы включите весь угловой момент, вам понадобится гораздо более продвинутая теория Эйлера, которая включает тензор инерции . Проблема с упрощением заключается в том, что во многих книгах не говорится, что их объяснение является приблизительным .

У него есть угловой момент в вертикальном направлении, так что это должно решить ваши проблемы. Существует горизонтальный крутящий момент, поэтому наклоненный вверх вектор углового момента просто описывает конус, где его скорость изменения всегда горизонтальна. Вертикальная составляющая обычно очень мала, но не представляет проблем, она просто остается постоянной.

Но взгляните на задачу 1.286. Здесь угловая скорость прецессии является функцией времени. Итак, здесь момент импульса должен иметь вертикальную составляющую, но происходит та же ситуация.

Нет, вы совершенно правы, если дело в вашем происхождении. О на рисунке выше, то единственный крутящий момент находится в тангенциальном направлении. Конечно есть угловая скорость. α в г направление. Как это работает, это радиальная составляющая углового момента л прецессы и г л г т "=" | л | α θ ^ , где θ ^ - единичный вектор в тангенциальном направлении. Приравнивая известный крутящий момент к г л г т теперь даст вам частоту прецессии α .

Это решение, на которое вы ссылаетесь, вероятно, имеет недостатки в обозначениях, потому что оно, кажется, предполагает изменение радиального углового момента за счет изменения г ю г т , что просто невозможно, так как не может быть чистого крутящего момента в радиальном направлении.

Интересно, что гироскоп улавливает небольшой постоянный угловой момент и в вертикальном направлении теперь, когда в начале угловой момент был чисто радиальным. Это означает, что подшипники вблизи точки контакта О создают тангенциальную силу и, следовательно, вертикальный крутящий момент, сначала в течение очень короткого периода времени, чтобы установить угловую скорость α гироскопа.

Но взгляните на задачу 1.286. Здесь угловая скорость прецессии является функцией времени. Итак, здесь момент импульса должен иметь вертикальную составляющую, но происходит та же ситуация.
Извините, у меня нет с собой книги. Я говорил о простейшем возможном случае равномерной прецессии, о чем, похоже, и был ваш вопрос.
Суть этой проблемы в том, что угловая скорость прецессии не равномерна, а является функцией времени. Мой (новый) вопрос: почему мы пренебрегаем вертикальной составляющей и в этом случае...?
Ни в коем случае нельзя пренебрегать вертикальной составляющей углового момента.

введите описание изображения здесьЭто анимация точного решения

введите описание изображения здесь

как вы можете видеть, у вас есть две дополнительные угловые скорости, ваше решение является устойчивым решением с   ϑ ˙ "=" 0   , ϑ "=" 0   и тензор инерции

я "=" [ м р 2 0 0 0 0 0 0 0 0 ]